RE: [obm-l] Problema OBM 3a fase

2009-04-13 Por tôpico marcone augusto araújo borges
...@hotmail.com To: obm-l@mat.puc-rio.br Subject: RE: [obm-l] Problema OBM 3a fase Date: Sun, 12 Apr 2009 16:50:58 -0300 Bem Marcone estava rabiscando um pouco, perdi uns minutinhos e consegui demostrar, a explicacao é muito facil, abaixo. Temos que para x2 + y2 + xy ser divisivel por 10

RE: [obm-l] Problema OBM 3a fase

2009-04-12 Por tôpico marcone augusto araújo borges
Oi,nehab,o problema correto é se xx+yy+xy é divisível por 10 então é divisível por 100.Desculpe e obrigado. Date: Sat, 11 Apr 2009 20:29:19 -0300 From: ne...@infolink.com.br To: obm-l@mat.puc-rio.br Subject: Re: [obm-l] Problema OBM 3a fase Oi, Marcone, Acho que este problema tem um quê

RE: [obm-l] Problema OBM 3a fase

2009-04-12 Por tôpico Joao Maldonado
Muito Obrigado Eric, até então eu desconhecia o teorema de Fermat. Vi que a demostracao do problema é bem facil. Grato. --- Em sáb, 11/4/09, Eric Campos Bastos Guedes fato...@hotmail.com escreveu: De: Eric Campos Bastos Guedes fato...@hotmail.com Assunto: RE: [obm-l] Problema OBM 3a fase Para

RE: [obm-l] Problema OBM 3a fase

2009-04-12 Por tôpico Joao Maldonado
Muito Obrigado Eric, até então eu desconhecia o teorema de Fermat. Vi que a demostracao do problema é bem facil. Grato. --- Em sáb, 11/4/09, Eric Campos Bastos Guedes fato...@hotmail.com escreveu: De: Eric Campos Bastos Guedes fato...@hotmail.com Assunto: RE: [obm-l] Problema OBM 3a fase Para

RE: [obm-l] Problema OBM 3a fase

2009-04-12 Por tôpico João Maldonado
...@hotmail.com To: obm-l@mat.puc-rio.br Subject: RE: [obm-l] Problema OBM 3a fase Date: Sun, 12 Apr 2009 13:10:46 + Oi,nehab,o problema correto é se xx+yy+xy é divisível por 10 então é divisível por 100.Desculpe e obrigado. Date: Sat, 11 Apr 2009 20:29:19 -0300 From: ne...@infolink.com.br

RE: [obm-l] Problema OBM 3a fase

2009-04-11 Por tôpico Eric Campos Bastos Guedes
Prove que existem infinitos inteiros positivos n tal que: 5^(n-2) -1 --- É um inteiro. n [1] Seja m um primo diferente de 5; [2] 5^(m-1) == 1 mod m (pelo Pequeno Teorema de Fermat) [3] 5^(2m-2) == 1 mod m (quadrando [2]) [4] 5^(2m-2) == 1 mod 2 (pois 5 eh impar) [5] m divide

RE: [obm-l] Problema OBM 3a fase

2009-04-11 Por tôpico marcone augusto araújo borges
alguem poderia resolver esse:Se x^2 +x*y + y^2 divide 10,então tbm divide 100 From: fato...@hotmail.com To: obm-l@mat.puc-rio.br Subject: RE: [obm-l] Problema OBM 3a fase Date: Sat, 11 Apr 2009 15:32:31 +0300 Prove que existem infinitos inteiros positivos n tal que: 5^(n-2) -1

Re: [obm-l] Problema OBM 3a fase

2009-04-11 Por tôpico Carlos Nehab
o problema deseja que se prove. Ento, ta uma possvel dica... Abraos, Nehab marcone augusto arajo borges escreveu: alguem poderia resolver esse:Se x^2 +x*y + y^2 divide 10,ento tbm divide 100 From: fato...@hotmail.com To: obm-l@mat.puc-rio.br Subject: RE: [obm-l] Problema OBM 3a fase Dat

RE: [obm-l] Problema OBM 3a fase

2009-04-11 Por tôpico Eric Campos Bastos Guedes
alguem poderia resolver esse:Se x^2 +x*y + y^2 divide 10,então tbm divide 100 Se xx + xy + yy divide 10, entao dividirah 100 tambem, pois todo divisor de 10 divide 100 [ ]'s E. - [ eric campos bastos guedes - matemático e educador

RE: [obm-l] Problema OBM 3a fase

2009-04-11 Por tôpico marcone augusto araújo borges
Desculpe Eric,mas o problema correto é :se x^2 + x*y +y^2 è divisível por 10 então é divisível por 100.Obrigado pela atenção.Um abraço. From: fato...@hotmail.com To: obm-l@mat.puc-rio.br Subject: RE: [obm-l] Problema OBM 3a fase Date: Sun, 12 Apr 2009 02:48:40 +0300 alguem poderia

RE: [obm-l] Problema de idades (socorro, estou apanhando muito nesse)

2009-03-27 Por tôpico marcone augusto araújo borges
Alô marcelo costa,marcone quer saber se vc recebeu a solução do problema das idades(de maria e joão)...só pra lembrar, a idade de joão foi dada(36 anos) e a de maria,pedida.UM abraço. Date: Mon, 23 Mar 2009 08:02:23 -0300 Subject: [obm-l] Problema de idades (socorro, estou apanhando muito

Re: [obm-l] Problema de idades (socorro, estou apanhando muito nesse)

2009-03-23 Por tôpico smolka
Hoje: idade de João = 36, idade de Maria = x. Em algum momento no passado: idade de João = x, idade de Maria = 18. Como a diferença entre as idades de João e Maria é constante, então: 36 - x = x - 18, logo x = 27 (idade de Maria hoje). [ ]'s JOãO TEM, HOJE, 36 ANOS, IDADE QUE é IGUAL

[obm-l] Re: [obm-l] Problema da EPCAR 1986... será erro no enunciado ou problema do operador?

2009-02-28 Por tôpico 111111
Rauryson, tentei refazer o exercício e cai nessa mesma resposta, deve ser erro do enunciado? Talvez, mas o que eu fiz e o que você fez está correto. Vamos esperar pra ver novas respostas. Amplexo 2009/2/27 Rauryson Alves raury...@yahoo.com.br ENUNCIADO: Um conjunto A tem m elementos e a

Re: [obm-l] problema análise

2009-01-08 Por tôpico silverratio
Olá, Começando pela letra a): Vamos assumir, sem perda de generalidade, que a sequência é crescente (estou admitindo aqui sequências constantes; talvez alguns prefiram o termo não-decrescentes). O outro caso é inteiramente análogo. Vou denotar o limite da subsequência convergente de {Xn} por L.

Re: [obm-l] Re: [obm-l] Problema 63 do Livro do Morgado - Geometria 1 - Página 111

2008-11-11 Por tôpico Marcelo Gomes
Valeu Fabrício, ok pode deixar vou melhorar da próxima vez...obrigado pelo site e a resolução do problema. Valeu, obrigado, abração, Marcelo. 2008/11/10 [EMAIL PROTECTED] [EMAIL PROTECTED] Em casos como este, tente enviar uma figura ou descrever o problema, para que mesmo os que não tenham

[obm-l] Re: [obm-l] Problema 63 do Livro do Morgado - Geometria 1 - Página 111

2008-11-09 Por tôpico [EMAIL PROTECTED]
Em casos como este, tente enviar uma figura ou descrever o problema, para que mesmo os que não tenham acesso ao livro possam participar. O problema é bastante conhecido, uma olhada no seguinte link deve ajudar: http://agutie.homestead.com/FiLEs/LangleyProblem.html É isso. Marcelo Gomes wrote:

Re: [obm-l] Problema de Mecânica

2008-10-25 Por tôpico Luiz Rodrigues
Olá Raphael, olá Davi.Muito obrigado pela ajuda.Eu também encontrei o problema no livro do Moysés.Abração!!!Luiz. 2008/10/22 Davi Costa [EMAIL PROTECTED]: É preciso utilizar o conceito de momento de inércia do cilindro, o momento de inércia é definido como o somatório do produto cada elemento

Re: [obm-l] Problema de Mecânica

2008-10-22 Por tôpico Davi Costa
É preciso utilizar o conceito de momento de inércia do cilindro, o momento de inércia é definido como o somatório do produto cada elemento de massa pela distancia ao seu eixo de rotação ao quadrado: I = Somatório(Mi*ri²) ou, em cálculo diferencial I = integral(r²,dm) No caso do cilindro temos

Re: [obm-l] Problema de Mecânica

2008-10-20 Por tôpico Bertoche Raphael Lydia
Bom, sou de ensino médio, mas no meu nível de mecânica não tenho errado: Pelo que entendi, são 2 corpos presos em cada ponta do fio, e a massa desse fio é desprezível. Pa=ma.g Pb=mb.g Como estão em mesma direção e sentidos contrários: F.R.=|Pa-Pb| F.R.=|ma-mb|*g Agora essa força moverá o sistema

Re: [obm-l] Problema - Campeonato Paulista

2008-08-21 Por tôpico Fernando Lima Gama Junior
20*19/2 = 190 2008/8/21 Dória [EMAIL PROTECTED] Olá! Podem me ajudar nesse exercício, por favor? No Campeonato Paulista de Futebol, participam 20 clubes. Se todas as equipes jogam entre si uma única vez, qual o total de partidas deste campeonato? [ ]'s

Re: [obm-l] Problema - Campeonato Paulista

2008-08-21 Por tôpico Dória
Quando faço 20*19 o que eu encontro? Obrigada. 2008/8/21 Fernando Lima Gama Junior [EMAIL PROTECTED] 20*19/2 = 190 2008/8/21 Dória [EMAIL PROTECTED] Olá! Podem me ajudar nesse exercício, por favor? No Campeonato Paulista de Futebol, participam 20 clubes. Se todas as equipes jogam

Re: [obm-l] Problema - Campeonato Paulista

2008-08-21 Por tôpico Iuri
Na primeira escolha vc tem 20 times pra escolher. Na segunda, tem 19, já que um deles foi escolhido anteriormente. Como escolher primeiro o time A e depois o time B ou primeiro escolher o time B e depois o time A são a mesma coisa, vc divide o resultado por 2. On Thu, Aug 21, 2008 at 7:18 PM,

Re: [obm-l] Problema - Campeonato Paulista

2008-08-21 Por tôpico Fernando Lima Gama Junior
Encontra todas as combinações, importando a ordem. Assim, tem que X - Y é diferente de Y - X. Quando se divide por 2, tem-se apenas uma combinação. Ou seja, para a primeira opção, temos 20 times. Para a segunda, 19. Assim, haveria 380 jogos (20x19) se os jogos fossem de ida e de volta. Como é

Re: [obm-l] Problema - Campeonato Paulista

2008-08-21 Por tôpico João Luís
- From: Dória To: obm-l@mat.puc-rio.br Sent: Thursday, August 21, 2008 7:18 PM Subject: Re: [obm-l] Problema - Campeonato Paulista Quando faço 20*19 o que eu encontro? Obrigada. 2008/8/21 Fernando Lima Gama Junior [EMAIL PROTECTED] 20*19/2 = 190 2008/8/21 Dória

Re: [obm-l] Problema - Campeonato Paulista

2008-08-21 Por tôpico Bruno França dos Reis
Vc encontra 380. Brincadeira, não resisti. Esse 20*19 é o que chamamos de *arranjo*. A(n, r) = arranjos de n elementos, tomados r a r = n! / (n - r)! O número A(n, r) é a quantidade de r-uplas (ordenadas) distintas que podemos formar a partir dos elementos de um conjunto de n elementos. No

Re: [obm-l] PROBLEMA DO CAVALO

2008-04-25 Por tôpico Henrique Rennó
Olá Maurizio! Parece que esse problema não é tão trivial não. Dá uma olhada nessa página abaixo com a descrição de uma solução. http://pessoal.sercomtel.com.br/matematica/alegria/burro/burro.htm Abraços! On Wed, Apr 23, 2008 at 7:20 PM, MauZ [EMAIL PROTECTED] wrote: Olá a todos, Um cavalo

Re: [obm-l] PROBLEMA DO CAVALO

2008-04-25 Por tôpico Arlane Manoel S Silva
Oi Maurício. Acho que estava enganado quanto ao resultado que dei. Errei nas contas. Desculpas. Tenho uma nova solução elementar. Dê uma olhada. http://www.linux.ime.usp.br/~arlane/elet.pdf inté, Citando MauZ [EMAIL PROTECTED]: Boa noite Arlane, Eu pensei um pouco sobre o problema

Re: [obm-l] PROBLEMA DO CAVALO

2008-04-25 Por tôpico MauZ
Obrigado! Adorei os links 2008/4/25 Arlane Manoel S Silva [EMAIL PROTECTED]: Oi Maurício. Acho que estava enganado quanto ao resultado que dei. Errei nas contas. Desculpas. Tenho uma nova solução elementar. Dê uma olhada.

Re: [obm-l] PROBLEMA DO CAVALO

2008-04-24 Por tôpico Arlane Manoel S Silva
Considere a circunferência de raio r e seja P pertencente a esta circunferência, o ponto onde está amarrada a tal corda de comprimento R, o qual devemos calcular. Agora considere a cirncuferencia de raio R centrada no ponto P. Então, a área entre as duas circunferências deve ser pi.r^2/2

Re: [obm-l] PROBLEMA DO CAVALO

2008-04-24 Por tôpico MauZ
Boa noite Arlane, Eu pensei um pouco sobre o problema e tudo que você disse eu pensei, o dificil pra mim foi realmente fazer as contas... Eu pensei também da seguinte forma: pego um semi circulo e fixo o seu ponto P numa extremidade, como se o corte fosse feito no diametro que contem P. Ai eu

Re: [obm-l] Problema envolvendo conjuntos

2008-04-04 Por tôpico Emanuel Valente
Olá João, você foi muito bem claro na explicação, valeu! Eu estava fazendo 3 diagramas, o terceiro era para as pessoas que não estudavam e nem praticavam esporte. Iria ficar todo o tempo do mundo e não iria conseguir achar a resposta. Até! On 4/4/08, João Luís [EMAIL PROTECTED] wrote: Emanuel,

Re: [obm-l] Problema Combinatória

2008-03-29 Por tôpico MauZ
Olá pessoal eu montei um esquema usando binomio de newton. Ficou da seguinte forma a resposta: Vou usar (a//b) como binomio a!/b!(a-b)! temos todas as combinações como: (n//p) e as que tem consecutivos como: (n-1//p-1)+(n-2//p-2)...(n-p+1//n-p), somando pela regra da diagonal: (n//p-1)-1 (sendo

Re: [obm-l] Problema Combinatória

2008-03-27 Por tôpico Johann Peter Gustav Lejeune Dirichlet
São 24 livros de assuntos distintos? E os livros estão grudados na estante (se o de Teoria da Computação está do lado de Linguagens Formais, eles sempre estarão lado a lado?) Bem, seria algo como escolher cinco números não-consecutivos do conjunto {1,2,3,4\ldots,24}. Acho que dá pra usar alguma

Re: [obm-l] Problema Combinatória

2008-03-27 Por tôpico Rogerio Ponce
Ola' Mauricio e colegas da lista, os 5 livros retirados determinam 6 intervalos , dos quais o mais 'a esquerda e o mais 'a direita podem ter o valor minimo de 0, e os outros quatro valem no minimo 1. Para uniformizar tudo, podemos imaginar que exista um livro a mais do lado direito, e outro do

Re: [obm-l] Problema das Vigas

2008-03-06 Por tôpico Joao Victor Brasil
AB=CD??? On 3/6/08, Eduardo Estrada [EMAIL PROTECTED] wrote: Olá, Alguém conhece uma solução simples para o Problema das Vigas? Consiste no seguinte: Imagine a seguinte figura: || A || |

Re: [obm-l] problema de cálculo

2008-01-21 Por tôpico Ralph Teixeira
Interpretacao geometrica: mostre que existe um ponto p na esfera |p|=c onde o gradiente de f eh normal aa dita esfera (portanto, paralelo a p) Dica para resolver o problema: o gradiente da f eh normal aa esfera meio que significa que a funcao f restrita aa esfera deve ter um ponto critico

Re: [obm-l] Problema com polinômios

2008-01-14 Por tôpico Marcelo Salhab Brogliato
Olá Igor, estou tentando encontrar um contra-exemplo (pra mim, é um ótimo jeito de se encontrar uma demonstração.. hehe!) p(x) = x^n + a_1*x^(n-1) + a_2*x^(n-2) + ... + a_(n-1)*x + a_n vamos supor que: p(a) = p(b) = p(c) = p(d) = 5, e p(k) = 8 onde a, b, c, d, k sao primos entre si dois a dois.

[obm-l] Re: [obm-l] Problema com polinômios

2008-01-14 Por tôpico Rafael Cano
Brogliato To: obm-l@mat.puc-rio.br Sent: Monday, January 14, 2008 9:22 AM Subject: Re: [obm-l] Problema com polinômios Olá Igor, estou tentando encontrar um contra-exemplo (pra mim, é um ótimo jeito de se encontrar uma demonstração.. hehe!) p(x) = x^n + a_1*x^(n-1) + a_2*x^(n-2

Re: [obm-l] Re: [obm-l] Problema com polin�mios

2008-01-14 Por tôpico Carlos Yuzo Shine
por que elas valem para os outros coeficientes? E esse método não vai acabar num coeficiente diferente de 1 para x^n? abraços - Original Message - From: Marcelo Salhab Brogliato To: obm-l@mat.puc-rio.br Sent: Monday, January 14, 2008 9:22 AM Subject: Re: [obm-l

Re: [obm-l] Problema

2007-11-20 Por tôpico saulo nilson
2 -35 - 48 2 -5- -3- x 1 -1 - -9- 45 T*(k1P+k2A)=N 3*(2k1+k2*5)=48 9(k1+k2)=45 2k1+5k2=16 2k1+2k2=10 k2=2 k1=3 N=5(6+6)=60 On 11/16/07, Antonio Manuel Castro del Rio [EMAIL PROTECTED] wrote: Alguém teria a solução do seguinte problema. Dois profissionais e cinco aprendizes, produzem 48

Re: [obm-l] Problema de contagem

2007-11-10 Por tôpico João Pedro de Gusmão Silva
João Pedro de Gusmão Silva [EMAIL PROTECTED] escreveu: Caro Henrique, em momento algum foi dito que os algarismos são distintos. A tua solução só é válida no caso em que os dígitos são distintos, no entanto, o problema pede todos os números possíveis, você deve considerar o caso em que os

Re: [obm-l] Problema de contagem

2007-11-04 Por tôpico João Pedro de Gusmão Silva
Caro Henrique, em momento algum foi dito que os algarismos são distintos. A tua solução só é válida no caso em que os dígitos são distintos, no entanto, o problema pede todos os números possíveis, você deve considerar o caso em que os dígitos podem se repetir. Mesmo assim, obrigado!!!

Re: [obm-l] Problema de contagem

2007-11-01 Por tôpico Henrique Rennó
Com os dígitos 1,2,3,4,5 e 6, quantos números de 6 algarismo podemos formar, nos quais o 1 e o 2 não ficam juntos? Pode-se calcular o total de números de 6 algarismos com 1,2,3,4,5,6 menos o total de números em que o 1,2 estão juntos. 6! -- números de 6 algarismos com 1,2,3,4,5,6 2!*5! --

Re: [obm-l] Problema!

2007-09-03 Por tôpico Rogerio Ponce
Ola' pessoal, inicialmente, durante M horas (correspondentes a meio dia) , N pessoas trabalharam no campo grande. Em seguida, durante M horas, N/2 pessoas ainda trabalharam no campo grande. Enquanto isso, no campo pequeno, N/2 pessoas trabalharam durante M horas, e, no dia seguinte, 1 pessoa

Re: [obm-l] Problema de matematica!

2007-08-29 Por tôpico Bruno França dos Reis
Chame de x a qtd retirada do primeiro barril e y a do segundo. Agora é só escrever exatamente o que diz o enunciado: x + y = 14 12/(12 + 18)*x + 9/(9+3)*y = 7 Agora vc resolve o sistema: x + y = 14 2/5 * x + 3/4 * y = 7 ... Abraço Bruno 2007/8/29, jose silva [EMAIL PROTECTED]: Se

Re: [obm-l] Problema de matematica!

2007-08-29 Por tôpico jose silva
Parabens, companheiro! Muito obrigado. Claudio. From: Bruno França dos Reis [EMAIL PROTECTED] Reply-To: obm-l@mat.puc-rio.br To: obm-l@mat.puc-rio.br Subject: Re: [obm-l] Problema de matematica! Date: Wed, 29 Aug 2007 20:59:48 +0200 Chame de x a qtd retirada do primeiro barril

Re: [obm-l] Problema de Geometria

2007-07-30 Por tôpico Rogerio Ponce
Ola' Benedito e colegas da lista, acho que deve ter um jeito mais simples de fazer isso, mas vamos la'... Inicialmente, vamos estabelecer um conceito e um teorema (que poderiam ser formalizados, mas o texto fica muito longo. Como e' quase intuitivo, vou apenas mostrar a ideia) Conceito: Em

[obm-l] Re: [obm-l] Problema nº 8 NIVEL 3 OBM 2007

2007-06-27 Por tôpico Fábio Jesus Moreira de Almeida
valeu rogerio From: Rogerio Ponce [EMAIL PROTECTED] Reply-To: obm-l@mat.puc-rio.br To: obm-l@mat.puc-rio.br Subject: Re: [obm-l] Problema nº 8 NIVEL 3 OBM 2007 Date: Wed, 27 Jun 2007 01:48:30 -0300 (ART) Ola' Fabio, numa PA generica temosAn = A1 + (n-1)*r Quando r=1 , entao

Re: [obm-l] Problema nº 8 NIVEL 3 OBM 2007

2007-06-26 Por tôpico Andre Araujo
Olá Fábio, o n^4 - 7 que aparece é o número de termos da PA (4, 5, 6, ..., n^4 - 6 , n^4 - 5, n^4 - 4) de razão 1. Basta usar a fórmula do termo geral: aN = a1 + (N -1)*r = n^4 - 4 = 4 + (N - 1) = N = n^4 - 7. André Araújo. Em 26/06/07, Fábio Jesus Moreira de Almeida [EMAIL PROTECTED]

Re: [obm-l] Problema nº 8 NIVEL 3 OBM 2007

2007-06-26 Por tôpico Adélman de Barros Villa Neto
Olá Fábio, o n^4 - 7 é o numero de elementos da sequencia n=n^4 - 4 -3 , -3 pq a sequencia já começa no 4 (1...23...4),então devemos tirar o 1, o 2 e o 3 Tue, 26 Jun 2007 16:16:31 +, Fábio Jesus Moreira de Almeida [EMAIL PROTECTED] escreveu: Olá, meu nome é Fábio, e gostaria

Re: [obm-l] Problema nº 8 NIVEL 3 OBM 20 07

2007-06-26 Por tôpico Felipe Sardinha
Bom dia Fábio. Eis aqui meu raciocínio: Seja S = 4/n^4 + 5/n^4 + 6/n^4 + ... + (n^4 - 5)/n^4 + (n^4 - 4)/n^4 Somando parcelas equidistantes, temos: 4/n^4 + (n^4 - 4)/n^4 = 1 5/n^4 + (n^4 - 5)/n^4 = 1 6/n^4 + (n^4 - 6)/n^4 = 1 ... (e assim, sucessivamente) Basta agora

Re: [obm-l] Problema do ourives

2007-06-26 Por tôpico Rogerio Ponce
Ola' Raphael, X gramas da primeira liga equivalem a 4 X / (4+3) gramas de ouro 3 X / (4+3) gramas de prata Y gramas da segunda liga equivalem a 2 Y / (2+5) gramas de ouro 5 Y / (2+5) gramas de prata Assim, a liga produzida tem (4X + 2Y) / 7 gramas de ouro (3X + 5Y) / 7 gramas de

[obm-l] Re: [obm-l] Problema nº 8 NIVEL 3 OBM 2007

2007-06-26 Por tôpico Fábio Jesus Moreira de Almeida
desta lista, estou abrindo uma lista na USP, de problemas e teoremas, divulgo em breve bom falar com vcs From: Felipe Sardinha [EMAIL PROTECTED] Reply-To: obm-l@mat.puc-rio.br To: obm-l@mat.puc-rio.br Subject: Re: [obm-l] Problema nº 8 NIVEL 3 OBM 2007 Date: Tue, 26 Jun 2007 14

[obm-l] Re: [obm-l] Problema nº 8 NIVEL 3 OBM 2007

2007-06-26 Por tôpico Fábio Jesus Moreira de Almeida
denominador barraria a expansão da fórmula From: Adélman de Barros Villa Neto [EMAIL PROTECTED] Reply-To: obm-l@mat.puc-rio.br To: obm-l@mat.puc-rio.br Subject: Re: [obm-l] Problema nº 8 NIVEL 3 OBM 2007 Date: Tue, 26 Jun 2007 14:10:06 -0300 Olá Fábio, o n^4 - 7 é o numero de elementos da

[obm-l] Re: [obm-l] Problema nº 8 NIVEL 3 OBM 2007

2007-06-26 Por tôpico Fábio Jesus Moreira de Almeida
OBRIGADO ANDRE, mas Sim, por PA sai. mas analiticamente toda PA tem uma lei de formação, que não pode ser SOMATORIA de (n^4-7)/n^4 From: Andre Araujo [EMAIL PROTECTED] Reply-To: obm-l@mat.puc-rio.br To: obm-l@mat.puc-rio.br Subject: Re: [obm-l] Problema nº 8 NIVEL 3 OBM 2007

RE: [obm-l] Problema do ourives

2007-06-26 Por tôpico Fábio Jesus Moreira de Almeida
OI RAPHAEL, por proporção, o OURIVES sai resolvido assim: na primeira liga x 4(ouro)/7(4ouro + 3prata) + 3(prata)/7(4ouro + 3prata) e na segunda liga Y 2(ouro)/7(2ouro+5prata) + 5(prata)/7(5prata+2ouro) assim temos: OURO = 4x/7 + 2y/7 = 3x/7 + 5y/7 = PRATA multiplicando em

Re: [obm-l] Problema nº 8 NIVEL 3 OBM 20 07

2007-06-26 Por tôpico Rogerio Ponce
Ola' Fabio, numa PA generica temosAn = A1 + (n-1)*r Quando r=1 , entao An = A1 + (n-1) de forma que o numero n de termos e' igual a An - A1 + 1 E, obviamente, o termo medio e' (A1 + An)/2 , de forma que a soma dos termos e' (A1+An) * (An-A1+1) / 2 Repare que a divisao por 2 nao tem

Re: [obm-l] problema do livro

2007-06-25 Por tôpico Luís Lopes
Oi Vanderlei, Pode. A resposta no livro está imprecisa. Fique com a solução apresentada na lista. Acho que do Rogério. Um abraço, Luís From: vandermath [EMAIL PROTECTED] Reply-To: obm-l@mat.puc-rio.br To: obm-l@mat.puc-rio.br Subject: Re: [obm-l] problema do livro Date: Wed, 20 Jun 2007 10

[obm-l] Re: [obm-l] problema do livro é divertido resolver problemas

2007-06-20 Por tôpico Luís Lopes
Sauda,c~oes, Oi Vanderlei, Não está dito mas supõe-se que se eu não conheço você então você também não me conhece. Talvez aí esteja a sua dúvida. Um abraço, Luís From: Bruno França dos Reis [EMAIL PROTECTED] Reply-To: obm-l@mat.puc-rio.br To: obm-l@mat.puc-rio.br Subject: Re: [obm-l

Re: [obm-l] problema do livro

2007-06-20 Por tôpico vandermath
~oes, Oi Vanderlei, Não está dito mas supõe-se que se eu não conheço você então você também não me conhece. Talvez aí esteja a sua dúvida. Um abraço, Luís From: Bruno França dos Reis Reply-To: obm-l@mat.puc-rio.br To: obm-l@mat.puc-rio.br Subject: Re: [obm-l] problema do livro é

Re: [obm-l] problema do livro

2007-06-20 Por tôpico Qwert Smith
-rio.br Subject: Re: [obm-l] problema do livro Date: Wed, 20 Jun 2007 10:54:41 -0300 Prezado Luis Lopes A minha dúvida é a seguinte: Não pode acontecer de uma pessoa ser desconhecida de todas, mas todas as outras conheceram pelo menos uma pessoa? Como se esse pessoa fosse um penetra da festa? Um

Re: [obm-l] problema do livro é divertido resolver problemas

2007-06-20 Por tôpico Rogerio Ponce
, Não está dito mas supõe-se que se eu não conheço você então você também não me conhece. Talvez aí esteja a sua dúvida. Um abraço, Luís From: Bruno França dos Reis Reply-To: obm-l@mat.puc-rio.br To: obm-l@mat.puc-rio.br Subject: Re: [obm-l] problema do livro é divertido resolver problemas Date

Re: [obm-l] problema do livro é divertido resolver problemas

2007-06-20 Por tôpico Rogerio Ponce
Reis Reply-To: obm-l@mat.puc-rio.br To: obm-l@mat.puc-rio.br Subject: Re: [obm-l] problema do livro é divertido resolver problemas Date: Wed, 13 Jun 2007 23:32:31 -0300 Talvez fosse legal vc colocar detalhes sobre esse problema... se não quem não conhece o livro terá que ir atrás dele pra

Re: [obm-l] problema do livro é divertido resolver problemas

2007-06-13 Por tôpico Bruno França dos Reis
Talvez fosse legal vc colocar detalhes sobre esse problema... se não quem não conhece o livro terá que ir atrás dele pra responder pra vc! 2007/6/13, vandermath [EMAIL PROTECTED]: Caros colegas da lista! Não entendi a explicação do problema número 14 (o teorema da amizade) do livro do Luís

Re: [obm-l] Problema 2 nível 1 - 1998

2007-05-28 Por tôpico ralonso
Olá Hugo. É só falta de treino. Ninguém é burro. Jacira consegue datilografar 20 páginas de um manuscrito em 4 horas e Joana o faz em 5 horas. Ainda restam 900 páginas do manuscrito para datilografar. Se as duas começarem a datilografar no mesmo instante essas páginas, quantas páginas

[obm-l] RE: [obm-l] Problema 2 nível 1 - 1998

2007-05-27 Por tôpico Alexandre Salim
Vou montar as funções horárias da quantidade de páginas. P' - Jacira P - Joana P'=5t P=4t Sabemos que P+P'=900 = 9t=900 = t=100 horas P=4t = P=400 páginas. R: Deverá pegar 400 páginas. Deve ser isso. Abraço. | Alexandre Salim | From: Hugo Canalli [EMAIL PROTECTED] Reply-To:

RE: [obm-l] Problema da Eureka 25

2007-05-22 Por tôpico luis arthur bighetti
alguem sabe onde eu baixo o lidski obrigado _ Verificador de Segurança do Windows Live OneCare: verifique já a segurança do seu PC! http://onecare.live.com/site/pt-br/default.htm

Re: [obm-l] Problema da Eureka 25

2007-05-18 Por tôpico Henrique Rennó
Olá Saulo! Não entendi. Você poderia explicar com mais detalhes? Se você também puder apontar onde errei na solução. Obrigado! On 5/18/07, saulo nilson [EMAIL PROTECTED] wrote: a1a2a,,,an nao precisa terminar em zero, ja que ele e multiplicado por 100 que e divisivel portodos os numeros xyi.

Re: [obm-l] Problema da Eureka 25

2007-05-18 Por tôpico Henrique Rennó
Acho que resolvi. Já que temos que achar o número a1a2...an00 que seja divisível por XY, onde 1 = X = 9 e 1 = Y = 4, e o número a1a2...an é divisível por 100, nos fatores de 100 temos 2,2,5,5, ou seja, de todos os números de dois dígitos que podemos formar com os fatores de 100 o único que

Re: [obm-l] Problema da Eureka 25

2007-05-17 Por tôpico saulo nilson
a1a2a,,,an nao precisa terminar em zero, ja que ele e multiplicado por 100 que e divisivel portodos os numeros xyi. um numero par em baixo, cancela com 100 ficando um outro nuymero em baixo. On 5/17/07, Henrique Rennó [EMAIL PROTECTED] wrote: Olá!!! Estou tentando resolver o segundo problema

Re: [obm-l] problema do elevador

2007-05-13 Por tôpico saulo nilson
nao e que esse livro e famoso , muita gente tem e e facil de achar, achei que vc nao sabia que tinha la, nao tem resluçao , so da a resposta final. On 5/13/07, fernandobarcel [EMAIL PROTECTED] wrote: Ué Saulo, então não é com resposta e tudo - tem apenas uma resposta. E qual é a resposta?

Re: [obm-l] problema do elevador

2007-05-13 Por tôpico Emanuel Valente
Olá, eu tenho o livro e a resposta está correta. Abraços, Emanuel Valente. fernandobarcel escreveu: Ué Saulo, então não é com resposta e tudo - tem apenas uma resposta. E qual é a resposta? Pensando bem, quase todos problemas daqui devem estar resolvidos em algum lugar. Mas saber disso não

Re: [obm-l] Problema de Desigualdade

2007-05-12 Por tôpico saulo nilson
Sejam *x*, *y*,* z* reais positivos tais que *xy* + *yz* + *zx* = 1. Prove que: 2x (1 - x²) + 2y (1 - y²) + 2z (1 - z²) x+ y+z (1+x²)² (1+y²)² (1+z²)² 1+x² 1+y² 1+z²] o problema equivalente a demonstrar que 2-2x^2=1+x^2 x=1/raiz3

Re: [obm-l] problema do elevador

2007-05-12 Por tôpico saulo nilson
esse problema e classico, tem no livro fundamentos da fisica com resposta e tudo, mas nao com resoluçao. On 5/5/07, Rogerio Ponce [EMAIL PROTECTED] wrote: Ola' Emanuel, Como a plataforma exerce uma forca Fn sobre o homem, entao o homem exerce uma forca igual e de sentido contrario no elevador.

Re: [obm-l] problema do elevador

2007-05-12 Por tôpico fernandobarcel
Ué Saulo, então não é com resposta e tudo - tem apenas uma resposta. E qual é a resposta? Pensando bem, quase todos problemas daqui devem estar resolvidos em algum lugar. Mas saber disso não adianta nada pra maioria de nós, a não ser que o lugar seja em algum site da internet, concorda?

Re: [obm-l] Problema de Desigualdade

2007-05-07 Por tôpico Lucas Prado Melo
Eu não entendi isso: tgA tgB + tgA tgC + tgB tgC = 1 - A+B+C = Pi/2 Poderia esclarer para mim, por favor? Em 06/05/07, charles[EMAIL PROTECTED] escreveu: Sejam x, y, z reais positivos tais que xy + yz + zx = 1. Prove que: 2x (1 - x²) + 2y (1 - y²) + 2z (1 - z²) x+ y+z

Re: [obm-l] Problema de Desigualdade

2007-05-06 Por tôpico charles
Sejam *x*, *y*,* z* reais positivos tais que *xy* + *yz* + *zx* = 1. Prove que: 2x (1 - x²) + 2y (1 - y²) + 2z (1 - z²) x+ y+ z (1+x²)² (1+y²)² (1+z²)² 1+x² 1+y² 1+z² De a função tangente ser bijetora no intervalo [0,pi/2], nos reais

Re: [obm-l] Problema de Desigualdade

2007-05-06 Por tôpico Marcelo Salhab Brogliato
Olá, se x 1/sqrt(3), y 1/sqrt(3), z 1/sqrt(3) ... xy 1/3 ... xz 1/3 ... yz 1/3 xy + xz + yz 1 ... opz! absurdo! entao, pelo menos 1 tem que ser menor ou igual a 1/sqrt(3)... se x 1/sqrt(3), y 1/sqrt(3), z 1/sqrt(3)... xy 1/3 .. xz 1/3 ... yz 1/3 ... xy + xz + yz 1 ... opz!

Re: [obm-l] Problema de Desigualdade

2007-05-06 Por tôpico Marcelo Salhab Brogliato
linda solucao!!! :) abracos, Salhab On 5/6/07, charles [EMAIL PROTECTED] wrote: Sejam x, y, z reais positivos tais que xy + yz + zx = 1. Prove que: 2x (1 - x²) + 2y (1 - y²) + 2z (1 - z²) x+ y+z (1+x²)² (1+y²)² (1+z²)² 1+x² 1+y² 1+z²

Re: [obm-l] problema do elevador

2007-05-05 Por tôpico Rogerio Ponce
Ola' Emanuel, Como a plataforma exerce uma forca Fn sobre o homem, entao o homem exerce uma forca igual e de sentido contrario no elevador. Portanto, sobre o elevador atuam as forcas Fn e M*g para baixo, e uma forca T (tensao na corda) para cima, cuja resultante acelera o elevador de a .

Re: [obm-l] Problema de Física

2007-04-08 Por tôpico Marcelo Salhab Brogliato
Ola, se ela sobe h, entao: v = (2*g*h)^(1/2) a) v = v0 + at ... v - v0 = at .. a = (v - v0)/t (vetorialmente) passando pra escalares, temos: a = (v + v0)/t b) a = [ 1,5 + sqrt(2*9,8*1,2) ] / 0,01 abracos, Salhab On 4/8/07, Diego Alex [EMAIL PROTECTED] wrote: Deixa-se cair uma bola da

Re: [obm-l] Problema de contagem

2007-04-08 Por tôpico Graciliano Antonio Damazo
Muito obrigado Hermann, eu entendi bem a sua suposta 'bagunça', estava bem explicado valeu e boa pascoa Tio Cabri st [EMAIL PROTECTED] escreveu: Graciliano, espero que vc entenda a minha bagunça: 720=2^4 3^2 5^1 (fatoração) 720= x y z (onde x=2^a1 3^b1 5^c1; y=2^a2

Re: [obm-l] Problema de Física

2007-04-08 Por tôpico Diego Alex
Salhab, agradeço o interesse, mas ainda restaram algumas dúvidas: 1° No gabarito é dado o valor de 1026. 2° Como vc chegou na equação da parte b. {a=[d1 + sqrt(2*g*d2)]/t_colis,solo} (suponho). Grato, Diego Em 08/04/07, Marcelo Salhab Brogliato [EMAIL PROTECTED] escreveu:

Re: [obm-l] Problema de Física

2007-04-08 Por tôpico Eduardo Wilner
O Marcelo deve ter se atrapalhado um pouco : em vez de d1 coloque a velocidade com que a bola chega ao solo ,i.e., novamente Torricelli - sqrt(2gd1)^(1/2) , já que vc. preferiu d a h. Obs: o problema devia ter especificado: aceleração média durante a colisão... Diego Alex [EMAIL PROTECTED]

Re: [obm-l] Problema de contagem

2007-04-06 Por tôpico Tio Cabri st
Graciliano, espero que vc entenda a minha bagunça: 720=2^4 3^2 5^1 (fatoração) 720= x y z (onde x=2^a1 3^b1 5^c1; y=2^a2 3^b2 5^c2; z=2^a3 3^b3 5^c3) temos que a1+a2+a3=4 (15 modos) b1+b2+b3=2 (6 modos) c1+c2+c3=1 (3modos) temos assim 15.6.3=270 modos de colocar xyz só que temos que

RE: [obm-l] Problema... Olimpiada Argentina

2007-03-27 Por tôpico Jorge Armando Rehn Casierra
Olá pessoal! Suponha que a quadrúpla ordenada que resolve 2^n=a^2+b^2+c^2+d^2, seja (a_n, b_n, c_n, d_n). Existem duas possibilidades para n natural, n ímpar e n par: Se n é ímpar, n=2k+1 (k=0), 2^n = 2^(2k+1) = 2*2^(2k) = (2^k)^2 + (2^k)^2 = 0 + 0 + (2^k)^2 + (2^k)^2, portanto a quádrupla que

Re:[obm-l] Problema... Olimpiada Argentina

2007-03-27 Por tôpico claudio.buffara
De:[EMAIL PROTECTED] Para:obm-l@mat.puc-rio.br Cópia: Data:Mon, 26 Mar 2007 21:45:28 -0300 Assunto:[obm-l] Problema... Olimpiada Argentina Não consigo resolver: Para cada número natural, n, n diferente de zero, determinar os inteiros a, b, c e d, 0=a=b=c=d, tais que 2^n=a^2+b^2+c^2+d^2.

RE: [obm-l] Problema... Olimpiada Argentina

2007-03-27 Por tôpico Rogerio Ponce
Dessa forma todas as quádruplas (a, b, c, d) ficam determinadas para todos os valores possíveis de n natural. Todas as quadruplas? Nao, nao ficam determinadas. Para n=8 (2^8 = a^2 + b^2 + c^2 + d^2) , por exemplo, ha' solucoes como (a,b,c,d) = (0, 0, 0, 2^4) ou

RE: [obm-l] Problema... Olimpiada Argentina

2007-03-27 Por tôpico Rogerio Ponce
Corrigindo meu email anterior: Dessa forma todas as quádruplas (a, b, c, d) ficam determinadas para todos os valores possíveis de n natural. Todas as quadruplas? Nao, nao ficam determinadas. Para n=8 (2^8 = a^2 + b^2 + c^2 + d^2) , por

Re:[obm-l] Problema de algebra vetorial.

2007-03-14 Por tôpico claudio.buffara
Oi, Ronaldo: Chame de u o vetor de coordenadas (x1,y1,z1) e de v o segundo vetor. Gire os seus eixos coordenados de modo que as coordenadas de u passem a ser (0,0,L) e as de v (L*sen(theta),0,L*cos(theta)). Ou seja, u gera o novo eixo z e {u,v} gera o novo plano xz. Ao girar v de um angulo phi

Re: [obm-l] Problema de algebra vetorial.

2007-03-14 Por tôpico Ronaldo Alonso
Oi Cláudio: Obrigado por sua resposta. Andei pesquisando na internet sobre esse problema e parece que não tem mesmo outro jeito de fazer a não ser aquele que você apresentou abaixo. Muito obrigado por sua contribuição! []s Ronaldo. On 3/14/07, claudio.buffara [EMAIL PROTECTED] wrote:

Re: [obm-l] Problema sobre volume

2007-02-07 Por tôpico Eduardo Wilner
Não verifiquei se é o único poliedro possível, no problema, nem, se for, como demonstrá-lo; mas lá vai. Consideremos o poliedro com dois lados opostos do quadrado formando com as bases menores dos trapézios duas arestas e as bases maiores destes coincidindo numa terceira aresta. Os dois

Re: [obm-l] Problema

2007-01-30 Por tôpico Marcelo Salhab Brogliato
Olá, cara, acredito eu q temos q seguir a seguinte linha de raciocinio... comecamos com 1, subimos de 9 em 9.. voltamos com 7, e assim por diante.. 1 10 19 28 37 46 39 32 25 18 11 4 13 22 31 40 49 42 35 28 21 14 7 16 fiz 24 numeros.. acredito que caso siga essa ideia, dê certo.. ou, suba

Re: [obm-l] Problema Legal

2007-01-24 Por tôpico Carlos Gomes
Que bonito Benéeste problema é um dos meus preferidos...vou tentar explicá-la para quem por acaso não a conheça Suponha que a tal transformação seja possível. Imagine que uma vez feita a decomposição do círculo original você pegue cada parte e pinte de branco as linhas cercam uma região

Re: [obm-l] Problema de teoria dos numeros

2006-12-05 Por tôpico Ronaldo Alonso
On 12/4/06, Paulo Santa Rita [EMAIL PROTECTED] wrote: Ola carissimo Artur e demais colegas desta lista ... OBM-L, Seja M um primo tal que M = (2^N) + 1 e suponhamos que N nao e potencia de 2. Neste caso N e da forma : (2^P)*i, onde P e um inteiro nao-negativo e i um impar maior que 1. Segue

RE: [obm-l] Problema de teoria dos numeros

2006-12-05 Por tôpico Paulo Santa Rita
possivel. A Matematica usada ate agora nao e importante Um Abracao Paulo santa Rita 3,1123,051206 Date: Tue, 5 Dec 2006 07:55:29 -0200 From: [EMAIL PROTECTED] To: obm-l@mat.puc-rio.br Subject: Re: [obm-l] Problema de teoria dos numeros On 12/4/06, Paulo Santa

Re: [obm-l] Problema Interessante

2006-12-04 Por tôpico Marcelo Cruz
Podemos fazer o seguinte. Primeiro inserimos as peças, anotando com um lápis ou giz em cada casa quantas inversões aquela casa deve sofrer, no estilo prisioneiro contando os dias na parede da cela. Depois de colocadas todas as peças, fazem-se as inversões. Após ter colocado todas as peças, antes

Re:[obm-l] Problema Interessante

2006-12-04 Por tôpico claudio\.buffara
Cada nova peça colocada causa um número de mudanças de cor igual ao número de peças adjacentes a ela. Como, ao final do jogo, o tabuleiro está totalmente preenchido, o número total de mudanças de cor ocorridas é igual ao número total de pares de peças adjacentes, o qual, por sua vez, é igual

RE: [obm-l] Problema de teoria dos numeros

2006-12-04 Por tôpico Paulo Santa Rita
Ola carissimo Artur e demaiscolegas desta lista ... OBM-L,Seja M um primo tal que M = (2^N) + 1 e suponhamos que N nao e potencia de 2. Neste caso N e da forma : (2^P)*i, onde P e um inteiro nao-negativo e i um impar maior que 1. Segue daqui que M = (2^A)^ i + 1 com A= 2^P . Fazendo 2^A = X

[obm-l] Re:[obm-l] Problema da Olimpiada Piauie nse de Matemática

2006-11-29 Por tôpico claudio\.buffara
-- Cabeçalho original --- De: [EMAIL PROTECTED] Para: obm-l@mat.puc-rio.br Cópia: Data: Tue, 28 Nov 2006 18:26:48 -0200 Assunto: [obm-l] Problema da Olimpiada Piauiense de Matemática Prove que a³/bc + b³/ac + c³/ab = a + b + c De uma olhada no enunciado original. Ele deve

<    1   2   3   4   5   6   7   8   9   10   >